Tiempo que tardan en colisionar dos partículas con cargas opuestas

Creo que se supone que este es un problema simple, pero tengo un problema al convertirlo en un problema de un solo cuerpo. Es unidimensional. +q y -q a una distancia d de distancia, se mantienen estacionarios y luego se sueltan en t=0. El potencial es V(x)=kq^2/x. Si lo convierto en un problema de un solo cuerpo, entonces m-->m/2, pero ¿cómo interpreto la nueva x? Ambas partículas se mueven una hacia la otra, por lo que recorren una distancia d/2 antes de chocar. Supongo que la ecuación relevante será t = metro 2 d r mi V ( X )

¿Qué conceptos me faltan? Creo que se supone que esto es muy fácil, pero no es para mí.

editar, entonces x es ahora la distancia relativa entre las dos partículas, por lo que debería ser como una partícula que recorre toda la distancia d ? Obtengo un valor negativo, pero ¿es eso aceptable? Algo como

t = metro 2 d 0 d k q 2 d X 1 d / X Y eso no me está dando una muy buena respuesta cuando lo calculo.

Espero que sus cálculos tengan en cuenta la fuerza propia de la reacción de radiación.
No chocarán. En grandes ocasiones, se repelerán exponencialmente entre sí .
Encontré esto physics.stackexchange.com/questions/54655/… , ¿esto explica mejor la situación?

Respuestas (1)

El signo negativo no tiene que ver con la electrostática, es un problema con un signo global que viene de la raíz cuadrada.

En un problema 1D, puedes escribir

t F t i = X i X F d X v ,   dónde   v 2 = 2 metro ( mi V ( X ) ) .
En el presente caso, sin embargo, con X decreciendo como t aumenta, necesita tomar el signo negativo para v = d X d t . Así, si las partículas están a una distancia d en el tiempo cero y chocan en el tiempo T , Deberías escribir
T = d 0 d X 2 metro ( mi V ( X ) ) > 0.
¡Fácil!

Encontré este problema similar publicado, con la respuesta para el momento. Mi enfoque debe ser incorrecto ya que no puedo encontrar esa respuesta: physics.stackexchange.com/questions/54655/…
Me parece bien tu trabajo. Todo lo que necesitas hacer es hacer la integral y tener en cuenta las constantes.
Busqué el wolframio integral y obtuve un logaritmo como respuesta, t = π d q 2 π ϵ 0 metro d debería ser la respuesta correcta, aunque no estoy seguro de cómo llegar allí.
Esté atento a las señales: V ( X ) < mi < 0 .
Estaba buscando una solución y el signo de mi raíz cuadrada es incorrecto, es negativo. Lo solucioné, pero ahora no estoy seguro de si tengo un factor sqrt (2) incorrecto. Gracias por la ayuda.